Download Case Studies Weekly Independent Group Study Sessions

Document related concepts

Prenatal nutrition wikipedia , lookup

Dental emergency wikipedia , lookup

Patient safety wikipedia , lookup

Midwifery wikipedia , lookup

Prenatal development wikipedia , lookup

Maternal physiological changes in pregnancy wikipedia , lookup

Medical ethics wikipedia , lookup

Electronic prescribing wikipedia , lookup

Menstruation wikipedia , lookup

Women's medicine in antiquity wikipedia , lookup

Dysprosody wikipedia , lookup

Obstetrics wikipedia , lookup

Fetal origins hypothesis wikipedia , lookup

Prenatal testing wikipedia , lookup

Childbirth wikipedia , lookup

Transcript
Acknow
wledgment CASE SSTUDIES For weeklly STUDEENT GRO
OUP STU
UDY SESSSIONS CLINICAL CASE
EXAMINATION
Pelvic exam/annual exam
At the end of this exercise the student will be able to:
Discuss the indications for pelvic exam
Understand the important components of the pelvic exam
Know the indications for additional exam of the anterior and posterior vaginal
wall
Describe the importance of the rectal/vaginal exam.
52-year-old female comes in for annual gynecologic exam. She stopped menstruating
two years ago and has experienced hot flashes and some insomnia. She, however, feels
she is doing well and is not interested in HRT. Menarche was at age 13, menses were
regular until last year prior to cessation of menses, but then she stopped and has had
no more bleeding since that time. She had two pregnancies and two vaginal births; her
children are 25 and 28.
Her ROS is negative except for occasional loss of a small amount of urine in her
aerobics class; if she is careful to empty her bladder prior to class, this isn’t a problem.
She is sexually active and needs to use a lubricant; however, sex is not painful.
PMH is negative except for an appendectomy and breast biopsy, which was negative.
She is married in a good relationship with no history of abuse and she exercises
3x/week in aerobics class or walks. She drinks a glass of wine with dinner and has
never smoked or used illicit drugs. She drinks 3 glasses of milk/day. Family history is
significant for mother with osteoporosis, sister with hypothyroidism and father with
elevated cholesterol.
Physical exam
Normal appearing female 120/70, weight 130, height 5’5”
Thyroid-WNL; Lungs- Clear; COR- RRR no murmur or gallops; Breast exam –
breasts are symmetrical with fibrocystic changes in the upper out section of each
breast; no distinct masses are noted with evidence of small scar on R breast; Abdomen
– soft non-tender; Ext genitalia – normal with evidence of midline scar consistent
with vagina – no lesions, slightly atrophic; Cervix appears to be WNL and Pap smear
obtained. The speculum is taken apart and the lower blade of the speculum is used by
placing in the vagina pressing downward to evaluate the anterior wall of the vagina.
There is evidence of small cystocele and urethrocele, which descends slightly with
Valsalva, but not to the introitus. There is no loss of urine during this maneuver.
Bimanual reveals an anteverted uterus with no adnexal masses palpate. In fact, the
adnexa are not well appreciated, recto-vaginal exam is negative, confirms the vaginal
exam and the stool is heme negative.
You assure the patient that her exam is WNL. You begin counseling her re: lifestyle
issues, encourage her to continue exercising regularly and perhaps add some weights
to further decrease risk of osteoporosis. You counsel her regarding her needs for 1200
mg of calcium/day. Additionally, she should consider either a sigmoidoscopy or
colonoscopy for colorectal screening. A mammogram is ordered, as well as screening
cholesterol and thyroid screen. She is counseled regarding her Kegel exercises and
need to keep bladder empty during exercise and to do Kegels to strengthen pelvic
floor. She is sent out with a slip for cholesterol screening, thyroid screening, urinalysis
and a set of three fecal occult blood cards.
DISCUSSION/TEACHING POINTS
The annual exam is more than just a breast exam and pelvic exam. It is the
opportunity to assess a patient’s overall health and health care needs. The same
exam/laboratory tests are not done on each patient. One needs to consider age and
risk factors when deciding what to do. This patient is 52-years-old; therefore, it is
important to include colorectal screening and counseling in the exam. She should also
have annual mammography, counsel regarding calcium intake and consider thyroid
screening.
Although SBE has not been shown to decrease morbidity or mortality from breast
cancer, one should inquire if the patient does SBE and offer to teach her if she doesn’t
know how to do this.
PE findings of urethrocele and mild cystocele are very common in older women who
have had children. This is best detected on physical exam using a Sims speculum and
having the patient Valsalva. This allows one to examine the movement and degree of
prolapse of the anterior and posterior vaginal wall, as well as the cervix. A
ureterocele, cystocele and rectocele do not necessarily require surgical repair. Mild
symptoms of stress urinary incontinence, such as this patient reports, need to be
addressed and usually respond well to behavioral measures, such as keeping the
bladder empty during exercise and minimizing caffeinated beverages and to pelvic
strengthening exercises or Kegels.
On pelvic exam, the adnexa were not well appreciated, which is normal in
postmenopausal women. Often the adnexa are not felt and, if they are palpated,
might raise concern and necessitate further evaluation if thought to be enlarged. The
rectal/vaginal is done to confirm the findings of the vaginal exam and is important in
this age patient to evaluate for rectal cancers. Approximately 10% of rectal cancers
can be detected with rectal exam. However, it is important to add fecal occult blood
testing, which should not only be done at the time of the exam, but the patient also
needs to do this at home on three consecutive bowel movements. This will increase
detection of all colorectal cancers.
CLINICAL CASE
EXAMINATION
Pelvic exam/annual exam
At the end of this exercise the student will be able to:
Discuss the indications for pelvic exam
Understand the important components of the pelvic exam
Know the indications for additional exam of the anterior and posterior vaginal
wall
Describe the importance of the rectal/vaginal exam.
52-year-old female comes in for annual gynecologic exam. She stopped menstruating
two years ago and has experienced hot flashes and some insomnia. She, however, feels
she is doing well and is not interested in HRT. Menarche was at age 13, menses were
regular until last year prior to cessation of menses, but then she stopped and has had
no more bleeding since that time. She had two pregnancies and two vaginal births; her
children are 25 and 28.
Her ROS is negative except for occasional loss of a small amount of urine in her
aerobics class; if she is careful to empty her bladder prior to class, this isn’t a problem.
She is sexually active and needs to use a lubricant; however, sex is not painful.
PMH is negative except for an appendectomy and breast biopsy, which was negative.
She is married in a good relationship with no history of abuse and she exercises
3x/week in aerobics class or walks. She drinks a glass of wine with dinner and has
never smoked or used illicit drugs. She drinks 3 glasses of milk/day. Family history is
significant for mother with osteoporosis, sister with hypothyroidism and father with
elevated cholesterol.
Physical exam
Normal appearing female 120/70, weight 130, height 5’5”
Thyroid-WNL; Lungs- Clear; COR- RRR no murmur or gallops; Breast exam –
breasts are symmetrical with fibrocystic changes in the upper out section of each
breast; no distinct masses are noted with evidence of small scar on R breast; Abdomen
– soft non-tender; Ext genitalia – normal with evidence of midline scar consistent
with vagina – no lesions, slightly atrophic; Cervix appears to be WNL and Pap smear
obtained. The speculum is taken apart and the lower blade of the speculum is used by
placing in the vagina pressing downward to evaluate the anterior wall of the vagina.
There is evidence of small cystocele and urethrocele, which descends slightly with
Valsalva, but not to the introitus. There is no loss of urine during this maneuver.
Bimanual reveals an anteverted uterus with no adnexal masses palpate. In fact, the
adnexa are not well appreciated, recto-vaginal exam is negative, confirms the vaginal
exam and the stool is heme negative.
You assure the patient that her exam is WNL. You begin counseling her re: lifestyle
issues, encourage her to continue exercising regularly and perhaps add some weights
to further decrease risk of osteoporosis. You counsel her regarding her needs for 1200
mg of calcium/day. Additionally, she should consider either a sigmoidoscopy or
colonoscopy for colorectal screening. A mammogram is ordered, as well as screening
cholesterol and thyroid screen. She is counseled regarding her Kegel exercises and
need to keep bladder empty during exercise and to do Kegels to strengthen pelvic
floor. She is sent out with a slip for cholesterol screening, thyroid screening, urinalysis
and a set of three fecal occult blood cards.
DISCUSSION/TEACHING POINTS
The annual exam is more than just a breast exam and pelvic exam. It is the
opportunity to assess a patient’s overall health and health care needs. The same
exam/laboratory tests are not done on each patient. One needs to consider age and
risk factors when deciding what to do. This patient is 52-years-old; therefore, it is
important to include colorectal screening and counseling in the exam. She should also
have annual mammography, counsel regarding calcium intake and consider thyroid
screening.
Although SBE has not been shown to decrease morbidity or mortality from breast
cancer, one should inquire if the patient does SBE and offer to teach her if she doesn’t
know how to do this.
PE findings of urethrocele and mild cystocele are very common in older women who
have had children. This is best detected on physical exam using a Sims speculum and
having the patient Valsalva. This allows one to examine the movement and degree of
prolapse of the anterior and posterior vaginal wall, as well as the cervix. A
ureterocele, cystocele and rectocele do not necessarily require surgical repair. Mild
symptoms of stress urinary incontinence, such as this patient reports, need to be
addressed and usually respond well to behavioral measures, such as keeping the
bladder empty during exercise and minimizing caffeinated beverages and to pelvic
strengthening exercises or Kegels.
On pelvic exam, the adnexa were not well appreciated, which is normal in
postmenopausal women. Often the adnexa are not felt and, if they are palpated,
might raise concern and necessitate further evaluation if thought to be enlarged. The
rectal/vaginal is done to confirm the findings of the vaginal exam and is important in
this age patient to evaluate for rectal cancers. Approximately 10% of rectal cancers
can be detected with rectal exam. However, it is important to add fecal occult blood
testing, which should not only be done at the time of the exam, but the patient also
needs to do this at home on three consecutive bowel movements. This will increase
detection of all colorectal cancers.
INTRAPARTUM CARE
Learning objectives
At the conclusion of this exercise, the student will be able to:
A. Describe the initial assessment of the laboring patient
B. Describe the stages and mechanism of normal labor and delivery
C. Understand and interpret methods of monitoring the mother and fetus
D. Describe management of normal delivery
E. Understand the indications for operative delivery
F. Describe immediate postpartum care of the mother
Martha is a 26-year-old G2P1 at 40 weeks gestation who comes to labor and delivery
noting several hours of frequent, painful uterine contractions. About an hour prior to her
arrival she noticed the start of intermittent leaking of fluid and passage of slightly bloodtinged mucus per vagina. She states that the baby is moving normally. In reviewing her
chart, you find that she had an elevated 1-hour glucose screen of 150 with a normal 3hour glucose tolerance test. She had an ultrasound at 17 weeks that revealed a male fetus
and was consistent with her last menstrual period dating. A vaginal culture at 36 weeks
revealed no group B streptococcus in the vagina. The remainder of her prenatal care was
uneventful. Her prior medical history is significant for a work related lumbar spine strain
and an appendectomy at age 12. She is allergic to penicillin.
Physical exam
Her blood pressure is 96/54, pulse 92/minute, respirations are 20/minute and temperature
is 98F oral. Fetal heart rate (FHR) is in the 150s with good variability, positive
accelerations and no decelerations. Contractions are noted on the external monitor every
2-3 minutes and make the patient very uncomfortable. The fetal back is palpable at the
right side of the maternal abdomen and the vertex is palpable through the maternal
abdomen just below her symphysis pubis. Cervical examination reveals 3cm dilation,
80% effacement and a station of –2. Fluid taken from the vagina is Nitrazine positive and
leaves a fern pattern upon drying. This confirms spontaneous rupture of membranes.
Two hours later, Martha has requested intravenous pain medication and she is given a
narcotic combined with an anti-emetic. Cervical exam at that time reveals 5cm dilation,
complete effacement and –1 station. FHR is reassuring. An hour later, the narcotic has
worn off and Martha is requesting an epidural. The anesthetist is alerted but, first, another
cervical exam is performed. This time, the cervix is 6cm dilated and the vertex is at 0
station. The epidural is placed and dosed, with great relief to Martha.
Two hours later, the epidural dose is beginning to wear off and the fetal heart rate shown
mild, intermittent variable decelerations. A cervical exam reveals 8 cm dilation and a +1
station. The epidural is re-dosed. Martha sleeps after this and awakens to the nurse
coming to evaluate a fetal heart rate deceleration to the 80s that lasted about a minute
before recovering to the baseline 150s. A cervical exam reveals that the cervix is now
completely dilated and the vertex is at +2 station. The patient is instructed in pushing
technique. With the assistance of her husband and a friend, she pushes to crowning and is
taken to the delivery room for delivery. She delivers a male with Apgar 9/9 over an intact
perineum. The placenta delivers normally and Martha is taken to the recovery room with
her new baby, both in stable condition.
Diagnosis
Normal labor and delivery
Teaching points
1. Labor is defined as progressive dilation and effacement of the cervix in response to
regular uterine contractions. False labor is defined as contractions at term that do
not result in cervical change. This can be a frustrating situation both for the
pregnant woman and her physician since these ineffective contractions can be quite
uncomfortable. They can be managed with fluids, bed rest, hypnotics or narcotics.
2. Initial assessment of the laboring patient involves evaluating the fetal heart rate,
fetal presentation, the condition of the cervix, and the timing and quality of uterine
contractions. Evaluation of maternal vital signs, current medical conditions and
current physical status are essential. Medical conditions that might affect labor or
delivery must be addressed, as must the patient’s prior labor and delivery
outcomes.
3. The Friedman curve is a graph plotting the progress of labor over time. The
Friedman curve plots cervical dilation and fetal descent over time and is a means of
determining whether or not labor is moving along normally. If labor is not
progressing, several interventions are employed. Oxytocin is used in the case of
inadequate contractions. Forceps or vacuum extraction are indicated in the event of
maternal exhaustion or for nonreassuring fetal heart tracing Cesarean delivery may
be needed if there is evidence of cephalopelvic disproportion.
4. Labor is divided into stages: 1st stage is from the onset of contractions and cervical
change to complete dilation; 2nd stage is from complete dilation to delivery of the
fetus; 3rd stage is from delivery of the fetus to delivery of the placenta; some
authors have described the first hour postpartum as the 4th stage, during which
there is a risk of postpartum hemorrhage.
5. The fetus descends through the maternal pelvis through various flexions and
rotations called the cardinal movements of labor. These include engagement, where
the leading bony part of the fetal vertex has reached the ischial spines and the
maternal sacrum is partially filled by the fetal head; descent, where the fetal vertex
is 1 or more centimeters below the ischial spines (+1 station or more); flexion of
the fetal head at the neck due to increased resistance against the maternal pelvis;
internal rotation, which occurs at the level of the ischial spines as the biparietal
diameter of the fetal head passes through the maternal mid pelvis; and extension,
where the fetal head extends and the head emerges from the vagina.
6. Immediately postpartum, the mother is observed for alterations in vital signs or
postpartum hemorrhage. Vaginal blood flow (lochia) is evaluated and the uterus is
massaged to maintain a contracted state. Usually, oxytocin is administered
postpartum to assure uterine contractility and consequent decreased blood loss.
FETAL SURVEILLANCE and EFR STRIPS FHR STRIP INTERPRETATION Clinical Context must be understood Strip‐ Comment on: = Contractions = FHR strip Rate BLV Periodic changes Accelerations‐ what is the definition what is the criteria for a reactive strip antepartum Decelerations Early, Late and Variables. What are these and ehat is their physiology Intrapartum: Strip evolution is an important concetpt ie a strip slowly turning for the worst and knowing which ones migh be at risk for the same… “under special fetal watch” and readiness for the next step and doing all you can to improve it ie fetal and maternal resusitation as need be. What is a BPP? What is its usefulness. CLINICAL CASE
NORMAL AND ABNORMAL UTERINE BLEEDING
A 41-year-old G2P0020 LMP=10 days ago presents with persistent heavy vaginal
bleeding. She denies dizziness, but complains of feeling weak and fatigued. Her cycles
have been heavy for a long time, but seem to be worsening over the last several
months. Her cycles come every 28-35 days and she bleeds for 7-10 days. She describes
bad cramps, passing clots and using 2 boxes of maxi pads each cycle. She is worried
about losing her job if the bleeding is not better controlled. She only gets designated
break times from the assembly line to use the bathroom. She takes Ibuprofen every 46 hours for cramps. She denies any bleeding disorders in the family. She uses condoms
for contraception. She also complains of a pressure sensation and increased urinary
frequency.
Allergies: None; Medications: Ibuprofen as needed
Ob-Gyn history
Menarche 13/cycles 28-35 days/ 7-10 days.
Normal pap smears. History of Gonorrhea once and treated
1 elective termination at 16-years-old and 1 miscarriage at 10 weeks, about 2 years
ago
Past medical history
None
Past surgical history
D & C for miscarriage; tonsils and adenoids as a child
Social history
Nonsmoker. Occasional alcohol. No drugs. Works at a factory for machine parts
assembly.
Family history
Hypertension in mother and father. Mother had 1 miscarriage and 3 sons. Her
brothers are healthy and one has sickle trait. Her paternal grandfather died of lung
cancer.
ROS
Negative, except as noted above.
Physical exam
VS: BP=130/88; Pulse= 110; Respirations= 18; Ht=5’6’; Wt=150 pounds
African-American women who appears pale and with bags under her eyes
HEENT: NC and AT
Lungs: clear to auscultation and percussion
CV: rapid rate, no murmurs
Breasts: Non-tender, no masses, no dimpling, retraction or discharge
Abdomen: Non-tender, No hepatomegaly, firm palpable mass in the lower abdomen
Extremities: Non-tender, no edema, 2+/= DTRs bilaterally
Pelvic exam: Normal external genitalia; moist and pink vagina with rugae and dark
blood in the vault; cervix is non-tender, no lesions, and nullipara; uterus is 16 weeks
size, firm, mobile, non-tender; adnexae: non-tender, no palpable masses
Laboratory/studies
Hbg: 9.0, HCT: 27%
HCG: negative
TSH: 3.5 uIU/mL (Normal range: 0.4-4.0)
Prolactin: 19 ng/dl (Normal range <20)
PT/PTT: normal
Urinalysis: negative for infection
Endometrial biopsy: Proliferative endometrium
Pelvic Ultrasound: Multiple myomas (intramural and submucosal in location),
Normal ovaries
Diagnoses
Menorrhagia
Anemia
Leiomyomatous uterus
Possible anovulation (when her cycles are greater than or equal to 35 days)
Treatment
This patient was treated with GnRH analog for three months. Her hemoglobin
increased to 12. She had some minor spotting during therapy. She complained of hot
flushes and irritability. Her follow-up examination at 2.5 months of therapy showed a
decrease in uterine size to 12-14 weeks size. Her repeat ultrasound confirmed these
findings. She was counseled regarding medical management with oral contraceptives,
progestins or continued GnRH analog with hormonal add-back. Given the presence of
submucosal myomas, it is likely that this treatment may not be effective in the long
run. However, she has had an optimal response thus far.
She was also counseled regarding surgical management. If she is interested in
maintaining her fertility, her options include: hysteroscopic resection of submucosal
myomas only or abdominal myomectomy. If fertility is not desired and she wants a
definitive therapy, then a hysterectomy is indicated. Risks and benefits for these
medications and surgeries were discussed. This patient is at increased risk for
requiring a blood transfusion if the bleeding recurs and is heavy, or if the bleeding is
significant during surgery. She will think about her options and decide over the next
week.
Teaching points
1. Leiomyomas occur with a high prevalence of 25-50% of women (Buttram and
Reiter). They are more common in the African-American population.
2. If this patient did not respond to the gonadotropin agonist therapy and the
bleeding worsened, she would have been a candidate for high dose oral
contraceptives, high-dose Premarin intravenously or a D&C to control her
bleeding. If none of these options were effective, a uterine artery embolization
or hypogastric artery ligation could be options prior to hysterectomy. In the
near future, other medical therapies may become standard. Currently,
gonadotropin-releasing hormone antagonists and progesterone antagonist
mifepristone (RU 486) are under investigation. Gene therapies may be
developed as we learn more about leiomyoma formation and growth.
3. After a myomectomy, the recurrence rate of leiomyomas ranges from 27-51%.
Approximately, 15% require another operative procedure. The incidence of reoperation is increased with multiple myomas (26%) as opposed to a single
myoma (11%).
4. It is important to rule out the other differential diagnoses in women with
leiomyomas, i.e. pregnancy with possible incomplete abortion or ectopic,
thyroid disease, endometrial cancer, etc). These disorders are also present in
these women and we cannot assume we have the correct diagnosis unless
tested.
CLINICAL CASES
CONTRACEPTION AND STERILIZATION
Case 1
The patient is a 35-year-old G5P4 black woman with BMI > 30 who presents with
complaints of severe right lower extremity pain. She reports she was in her usual state of
health until about 5 days ago when she had onset of pain in her right lower extremity.
The pain has progressively worsened especially in her calf over the past five days. She
also has swelling, warmth and redness along her right lower extremity from the foot to
mid-thigh. She reports no headaches, dizziness, chest pain, shortness of breath, cough, or
dyspnea.
Ob-Gyn history
1.
2.
3.
4.
5.
Spontaneous vaginal delivery times 4
Elective abortion
Started OCP (Ortho Novum 7-7-7) about 1 year ago
LMP 1 year ago
Negative Pap smear & STD screen
Past medical history
1. Varicose veins
2. Pneumonia at the age 12
Past surgical history
1. Cholecystectomy at age 30
2. Tonsillectomy at age 13
Social history
1. H/o tobacco use times ½-pack per day for 10 years; quit 1 year ago
2. No ETOH; no IVDA
3. Works as a secretary for a paper company
Allergy and medication
None
Family History
No cancers, DM, CAD, CVA, or HTN, positive for DVT in her mother
Physical Exam
General: Obese black female in mild distress from leg pain.
VS BP 130/80, RR18, P 86 and regular; wt: 243
HEENT: PERRLA, NC, NT
Chest: clear to auscultation and percussion
Cardiovascular: Normal rate and rhythm, no murmurs
Breasts: No masses, adenopathy or skin changes
Abdomen: No hepatosplenomegaly, non-tender, obese
Pelvic: External genitalia: Normal
Vagina: Moist, pink, no discharge
Cervix: Parous, no lesions
Biman: Small, anteverted non-tender, no adnexal masses
Extremities: Right lower extremity with posterior calf tenderness, warmth,
swelling, and increased pain during dorsiflexion of the foot.
Laboratory or studies
Hbg 10.8 Hct 31.7 vol.%
PT/ PTT 12 sec/ 55 sec
Protein S - pending
Protein C - pending
Von Willebrands - pending
Lupus Anticoagulant - pending
A duplex venous ultrasonography of her right lower extremity showed occlusion
of the popliteal vein.
Diagnosis
Deep vein thrombosis
Treatment
The patient was immediately placed on heparin anticoagulation. She was also started on
coumadin and heparin was discontinued once she achieved therapeutic levels of
coumadin. She was placed on oral anticoagulation for 6 months, and oral contraceptives
were discontinued. She was offered a Paragard IUD vs. a tubal ligation.
Teaching points
1. Combination oral contraception is contraindicated in women who have a history
of idiopathic venous thromboembolism. In women who have a family history
(this patient’s mother had a DVT), the World Health Organization gives a
Category 2 rating: A condition where the advantages of using the method
generally outweigh the theoretical or proven risks. Although the patient’s family
history was not a contraindication to oral contraceptives, her personal history of a
DVT now presents a contraindication.
2. The rare woman on oral contraception who has a thrombotic episode may have an
underlying clotting problem, such as abnormality of Factor V in the clotting
cascade (Reference 2.) This patient is likely to have some kind of clotting factor
deficiency because her mother has a history of DVT. She should undergo testing
for these abnormalities.
3. Risk factors for venous disease include (Reference 3.)
a. BMI > 30
b. Immobility
c. Excessive varicosities
d. Family history of DVT in first-degree relative under 45
4. A non-hormonal IUD or sterilization are optimal choices of contraception in this
patient.
5. Combined oral contraceptives are very safe in the general population. Good
history-taking and adequate counseling may reduce the complications of oral
contraceptives (Reference 3.)
Case 2
The patient is a 37-year-old G3P2 who presents with dysfunctional uterine bleeding—
heavy irregular menses for the last 8 months. She has been using progestin-only oral
contraceptive pills for contraception. She had previously been taking combined oral
contraceptives but was switched to progestin-only pills 1 year ago because her physician
told her that her cardiovascular risk was increased because smokes a pack a day of
cigarettes. She presents to discuss contraceptive options. Her past medical history is
unremarkable except for mild hypertension treated with maxide, 25 mg PO q day. She
has been in a mutually monogamous relationship for 13 years.
Physical examination
General: well nourished and well developed
Ht: 5 feet 4 inches
Wt: 128 pounds
BP: 130/84
HEENT: WNL, no thyromegaly or adenopathy
Breasts: No masses adenopathy or skin changes
Heart: normal S1 and S2
Lungs: clear to auscultation
Abdomen: Soft, no masses, non-tender
Pelvic: External genitalia: normal
Vagina: pink and moist; mild active bleeding
Uterus: small, anteverted, non-tender, no adnexal masses
Lab work
The laboratory work done shows:
CBC
Hematocrit
32
Hemoglobin 10
WBC
9000
HCG -- less than 2mIU/ml
Cervical cultures – Gonorrhea and Chlamydia cultures negative
Endometrial biopsy - negative
Considerations
Contraceptive options in a patient over 35 who smokes, who is experiencing
dysfunctional bleeding and has a normal pelvic exam. Options include Mirena
IUD, sterilization, barriers
Teaching points
1.
Mirena IUD may be ideal for this patient
•
Highly effective contraceptive
•
Single act of motivation, long duration of action(5 years)
•
Reduction of menstrual bleeding will also help with this patient’s
dysfunctional uterine bleeding
2.
Sterilization: Counseling should include:
•
The permanent nature of the procedure
•
Availability of effective, long-term reversible contraceptives
•
Operative risks
•
Failure rate
3.
Barrier methods
•
Diaphragm: advantages, disadvantages
•
Condoms: advantages, disadvantages
4.
Hormonal contraceptives
•
Discussion of contraindications to combined oral contraceptives.
•
Discussion of advantages, disadvantages of progestin-only
contraceptive pills.
CLINICAL CASE
CLIMACTERIC
A 48-year-old female G3P3 whose last menstrual period was 6 months ago presents to
your office complaining of hot flushes, emotional liability and insomnia. She has tried to
use an over-the-counter soy supplement, but her symptoms are unchanged.
She began her menses at age 14. Her periods have been regular until 2 years ago. At that
time, they began to space out to every 2-3 months. She denies intermenstrual bleeding.
She had a negative Pap test one year ago and has never had an abnormal Pap. She is
sexually active and recently has noted some dyspareunia. Her husband had a vasectomy
15 years ago.
Her past medical and surgical history is significant only for an appendectomy at age 18.
She has smoked one pack of cigarettes per day for 30 years. Her family history is
significant for a mother that fractured her hip at age 83. She takes no medications or
supplements on a regular basis.
Physical exam
Slightly obese white female. Weight 180 lbs., 5’6” BP 126/76. General physical exam,
including breast exam within normal limits. Pelvic exam shows normal external genitalia.
Her vagina is slightly atrophic with decreased rugae. Her cervix is normal in appearance;
uterus is small and antiverted. Neither ovary is palpated on bimanual or rectal exam. Her
stool is guiac negative.
Laboratory
Pap smear – normal squamous epithelial and endocervical cells present.
Cholesterol 195 TSH 2.0
FSH 80 MIU/ml
Mammogram – negative
Differential Diagnosis
Menopausal vasomotor symptoms
Management plan
Encourage patient to stop smoking. Initiate incorporation of approximately 1200 mg
calcium and 400 units of vitamin D in her diet. After lengthy discussion of risks and
benefits of hormone therapy (HT), the patient decided to proceed with treatment, and was
given estrogen and progesterone in a continuous daily fashion.
1. Cigarette smoking is the largest preventable cause of death and disability among
women in the United States. Among women of reproductive age, 29% smoke.
Several studies have shown that smokers cease menstruating 1-2 years earlier than
non-smokers. This effect is dose dependent and the difference persists after
controlling for the subject’s weight. Moreover, female smokers 60 years of age
and older have significantly reduced bone mineral density of the hip compared
with nonsmokers.
2. With the onset of menopause, ovarian production of estrogen is significantly
reduced, lending to physiologic changes including hot flushes, mood disturbances,
sleep difficulty, thinning of genitourinary tissues, dyspareunia, metabolic shift to
a more atherogenic lipoprotein profile and accelerated loss of bone mass. This
transition in a woman’s life often is a good time to assess current health risks and
plan risk reduction strategies.
3. Estrogen/progestin hormone therapy is approved by the FDA for the treatment of
menopausal hot flushes, prevention of osteoporosis and treatment of urogenital
atrophy. Hormone therapy is the most effective agent known for the treatment of
hot flushes. Studies have also shown it to be significantly more effective than
placebo in reducing insomnia, irritability and poor short-term memory during the
menopausal transition.
4. Risks associated with hormone therapy include increased thromboembolic events,
increased triglyceride levels, breast cancer, gall bladder disease and other possible
conditions. It is reasonable to continue hormone therapy for the shortest amount
of time to address current vasomotor symptoms in an attempt to decrease overall
risk.
5. This patient’s risk for osteoporosis is increased by her mother’s history of hip
fracture. In addition to the protection provided by her HT, she should be
encouraged to maintain an appropriate daily intake of calcium and vitamin D.
Weight training exercise will also decrease her risk of fracture.
6. The patient should be encouraged to use a water-based lubricant to decrease
discomfort during sexual intercourse. The hormone therapy she has initiated
should improve the quality of her urogenital tissues over the next few months. If
these changes are not sufficient, the addition of a vaginal estrogen product may be
helpful.
Case Study: URINARY INCONTINENCE
Q: What is the mechanism of continence- urethral dominance- what promotes it?
Q: What the different types of incontinence?
Q: What is key urinary history?
Q: What is office cystometry- Ask woman to void, check residual and then fill bladder via
catheter attached to a empty syringe sleeve- watch meniscus- Why? Also ask for first sensation
and full bladder- why?
History
Ms. EQ is a 56 y/o Caucasian female with a six month history of acute onset urge-related urinary
incontinence. She denied dysuria, hematuria, sense of fecal urgency or fecal incontinence. Previous
urinalysis revealed no evidence of bladder infection.She has no ongoing medical illnesses and does
not take medication on a regular basis. She denies any global or localizing neurological symptoms.
She had three C-sections and a hysterectomy remotely for localized uterine cancer. She has not
been on hormone replacement therapy and is not currently sexually active.
She is employed in sales and her job involves extensive driving. She drinks coffee and caffeinated
sodas to stay alert. She rarely drinks more than sips of water. She has never used tobacco products.
Exam
Her urinalysis is negative for blood and evidence of infection. Ultrasound estimation of post-void
residual urine was negligible.
Abdominal examination reveals a moderately obese abdomen with active bowel sounds and no
palpable masses. Vaginal exam revealed atrophic external genitalia and vaginal tissues. She had
good anatomic support of her bladder and urethra and no evidence of enterocele, vault prolapse, or
rectocele. She was able to perform a weak and unsustained voluntary contraction of pelvic floor
muscles. Her bulbocavernosus reflex was intact. Anal tone was normal.
Her screening neurological examination (perineal sensation, anal tone, lower extremity sensation
and reflexes) was unremarkable.
Discussion
Acute onset of urge-related incontinence can be secondary to a bladder infection which are often,
but not always associated with symptoms of frequency and dysuria. As the urinalysis shows no
evidence of infection, her history of acute onset of urge incontinence should raise suspicion for
neurogenic etiology. In the absence of any history or physical findings supporting a neurological
event, there is no need to immediately pursue an extensive neurological assessment. The absence
of blood in the urine and the fact that she is a nonsmoker makes it unlikely that there is an
underlying bladder malignancy causing the symptoms. She has no post-void residual making a
bladder emptying dysfunction unlikely.
Once the “serious” potential causes are eliminated, attention can be turned to other potential causes
and contributing factors. Her behavior of drinking significant amounts of caffeinated coffee and colas
can be an underlying source of these symptoms, especially when combined with atrophic urogenital
tissues. It is also important to know what her “safe” bladder capacity is - volumes voided when she
gets to the toilet in time.
Plan 1
She was asked to keep a “Bladder Diary” - recording the time and volume of urinations; increase her
water intake; and decrease her intake of coffee and soda and return in one week to go over her
diary.
ONE WEEK FOLLOW-UP
Her daytime bladder volumes are 200-500cc and nighttime volumes - 200-300cc. She had noticed
less urgency with increased water intake.
Plan 2
Begin topical estrogen and low dose anticholinergic each morning and follow-up in six weeks.
Six Week Follow Up
Her frequency and urge incontinence almost resolved. She had dramatically decreased her coffee
and soda while increasing her water. No problems with topical estrogen. She had not filled her
prescription for the anticholinergics. Her symptoms were completely managed with topical estrogen
replacement and behavioral changes in her fluid intake. It is important to note that decaffeinated
coffee and sodas can also be “irritating” to the bladder - the acidity is an independent factor to the
caffeine.
List of Potential Bladder Irritants
(acidic and high potassium foods) ACIDIC FOODS AND BEVERAGES
CLINICAL CASE
GYNECOLOGIC PROCEDURES
A 25-year-old woman G3P2012 presents for counseling following an abnormal Pap
smear. The result of her most recent Pap smear was low-grade squamous
intraepithelial lesion (LGSIL). She has had a prior abnormal Pap smear about one
year ago and it also was low-grade SIL. She did not return to the office between these
two Pap smears. This time, a HPV typing screen revealed the presence of high-risk
groups.
Allergies and medications
None
Ob-gyn history
LMP= 1 week ago. Menarche 12/ Cycles 28 days/ 5 days duration. Contraception:
None
Sexually active with current boyfriend. She has had 10 sexual partners in her lifetime.
STDs: Gonorrhea x 1, Chlamydia x 1, and H/O herpes. No episodes of PID.
2 full-term vaginal deliveries and 1 elective termination without complication.
Past medical history
None
Past surgical history
None
Social history
½ ppd tobacco for 9 years. Social alcohol drinker. No drugs. She doesn’t work. Her
two children are 8 and 5-years-old. They live with she and her boyfriend.
Family history
Noncontributory.
Physical exam
VS: stable. Normal examination
Laboratory or other studies
None. May consider a pregnancy test if the LMP was not normal.
Diagnosis
Abnormal Pap smear
Treatment
This patient had counseling regarding the diagnosis of low-grade squamous
intraepithelial lesion (LGSIL) with high risk HPV typing. She was counseled
regarding the colposcopy procedure with cervical biopsies and endocervical curettings.
The procedure was described with its risks and benefits. She decided to proceed with
the colposcopy. The entire transformation zone was visualized. Multiple areas of
acetol-white lesions were present and biopsies were taken. There were no abnormal
vessels, punctate lesions or mosaicism seen. These biopsies and endocervical curettage
were sent to pathology.
She was also counseled regarding the need to have close follow-up after this procedure
to discuss the results and potential additional therapy, i.e. observation, loop
electrosurgical excision procedure or LEEP, Cone biopsy, cryotherapy, or
hysterectomy. In any case, she will need close follow-up with more frequent Pap
smears. Results are pending.
Teaching points
1. The appropriate initial management for a patient with LGSIL is colposcopy,
NOT observation.
2. After a negative but adequate colposcopy, a patient may be followed with
more frequent Pap smears and possibly HPV typing. Positive high risk HPV
typing increases the risk of a more significant cervical neoplasm.
3. An adequate colposcopy means that the entire transformation zone was
visualized.
4. Her sexual activity without protection, multiple partners, prior STDs and
smoking habits put her also at increased risk for cervical neoplasms and
additional sexually transmitted diseases.
5. This patient should be offered repeat screening for the full panel of sexually
transmitted diseases, i.e. HIV, hepatitis screen, GC, Chlamydia, and RPR. In
addition, counseling regarding protection from STDs and contraceptive use
should be addressed in this situation.
6. Smoking cessation counseling should also be addressed; if not in the first
session, then in a subsequent session. Physician counseling impacts the success
of smoking cessation. She is at high risk for lifetime of smoking and increased
lung cancer or other related neoplasm risk.
7. The student should also take this opportunity to become familiar with the
procedures, risks and benefits of LEEP, cone biopsy and cryotherapy. If the
endocervical curettings are positive for dysplasia or a high-grade lesion is
identified, then a further diagnostic procedure like LEEP or cone biopsy are
warranted. If the ECC is negative, but dysplasia is present at the
mild/moderate level, then two options exist: 1) close observation with Pap
smears every 3-6 months for 3 years and annually thereafter; or 2)
cryotherapy. If the patient’s disease persists or progresses, LEEP procedure
would be warranted. The cone biopsy would be an alternative; however, the
LEEP is preferred because it removes less cervical tissue and can be performed
in an out-patient/office setting, which reduces cost considerably.
CLINICAL CASE
PREECLAMPSIA-ECLAMPSIA SYNDROME
Learning Objectives
At the conclusion of this exercise, the student will be able to:
1.
2.
3.
4.
Define preeclampsia/eclampsia
Discuss the symptoms, physical findings, and diagnostic methods.
Discuss the approach to management
Discuss potential maternal and fetal complications.
An 18-year-old nullipara with no prenatal care was brought to the ER after having
“two fits” at home. Her menstrual history indicated her to be at 36 weeks’ gestation.
Further history indicated that she had gained a lot of weight during the pregnancy,
that her face and hands had become puffy within the preceding 3 days, and that she
had complained of headache, blurred vision and abdominal pain earlier in the day.
Her past medical history was unremarkable.
Examination showed a comatose pregnant young woman with foamy spittle at the
edge of her mouth. Her blood pressure was 160/110, pulse 118/M, and her
temperature was 99.6ºF. The chest examination was unremarkable. Examination of
the gravid abdomen showed a fundal height consistent with dates, a single fetus,
vertex and engaged, with an estimated fetal weight of 3,000 gm and a fetal heart rate
of 136 beats per minute. There was generalized edema present. Deep tendon reflexes
were 4+/4+ with sustained knee and ankle clonus. Vaginal exam showed the cervix 12 cm dilated and 75% effaced. The vertex was presenting at 0 station. Membranes
were intact.
Laboratory evaluation showed 3+ proteinuria, a hematocrit of 45%, a BUN of 14
mg/100 ml, a plasma uric acid of 8.6 mg/100 ml, a serum creatinine of 1.0 mg/100 ml,
and normal electrolytes and coagulation studies.
She progresses to complete and +2 station. She has a 1-hour second stage and you
assist her delivery. After delivery, she has heavy bleeding.
Teaching points
1.
2.
3.
4.
Preeclampsia
a.
Risk factors/epidemiology
b.
Diagnosis
Mild/severe preeclampsia
a.
Eclampsia
b.
HELLP syndrome
Management
a.
MgS04
b.
Delivery
Complications
a.
Abruption
b.
IUGR/oligohydramnios
c.
DIC
d.
Uterine atony
e.
Postpartum hemorrhage
CLINICAL CASE
ENDOMETRIOSIS
A 28-year-old female presents complaining of the inability to conceive for the past
two years. She has never used oral contraceptives; she and her husband have not used
any form of birth control for over two years. Her menarche occurred at the age of 12
and her menses became very painful in her late teens. Since, she has had chronic
cyclical pelvic pain, which has been getting progressively worse over the years. This
pain is incapacitating at time and is usually located in multiple areas (midline
abdomen, pelvis and lower back). In addition to the pain, her menstrual periods have
become increasingly frequent and heavy. She experiences pain with intercourse that
has progressively gotten worse. She denies any non-cyclical vaginal bleeding,
discharge and weight loss. She states that her 22-year-old younger sister has always
had very painful menses.
Physical exam
4mm hyperpigmented, raised, non-tender nodule in the umbilical area. The pelvic
exam showed a fixed, retroverted uterus. The uterosacral ligaments on both sides were
nodular and mildly tender. A right adnexal mass was palpated and tender. It was
round and approximately 5 cm in diameter in the area of the right ovary. The left
ovary was slightly enlarged.
LABS
CBC, electrolytes, UA, and TFT’s were normal. Semen analysis revealed 2 ml of
semen, >40 million sperm per mL, 70% normal forms and 70% motile. Ultrasound of
the abdomen showed an ectogenic cystic mass in the right ovary, which measured 4
cm in diameter. The left ovary was reported as normal.
Diagnosis
Probable endometriosis with an ovarian endometrium
Management
Laparoscopy showed an endometrioma of the right ovary and focal implants on the
left ovary, which were described as bluish-gray “powder burns.” There were also
similar lesions ranging from several millimeters to 2cm on the uterosacral ligaments
bilaterally and the anterior cul-de-sac. There were dense adhesions on a portion of the
left fallopian tubes and filmy adhesions on the right tube. The endometrioma of the
right ovary was resected and the focal lesions were resected or destroyed using
electrocautery.
Teaching points
The appropriate treatment for endometriosis varies widely because of the spectrum of
clinical symptoms and the vast differences in the extent of the disease. Treatments,
whether expectant, medical or surgical, should be individualized depending on the
patient’s symptoms, extent of disease, desire for fertility and other factors. One
should consider a laparoscopy in patients to establish the nature and extent, as well as
potentially treat, the endometriosis before therapy.
The revised American Fertility Society’s (AFS) staging system is generally used to
stage endometriosis in the infertile patient. In the AFS system, points are assigned for
size and depth of implants and for the severity of adhesions in various locations.
Stages I through IV are assigned on the basis of points. Management of endometriosis
can be guided by the stage of disease and the desire for fertility.
Endometrial implants alone may not be enough to cause infertility. A comprehensive
cohort study of infertile women showed that endometriosis without adhesions did not
alter the cumulative conception rate.
The increasing appearance of endometriosis (macroscopic implants) in fertile
asymptomatic women has led to the suggestion that endometrial implants in the
pelvis are, to some extent, to be considered as a physiologic process in menstruating
women. The crucial question is at which stage does endometriosis become pathologic
and a cause of symptoms so that treatment is indicated. One hypothesis is that
endometriosis is a physiological process until recurrent bleeding develops in the
ectopic implants, at which time the lesions show progression causing symptoms.
CLINICAL CASE
ABORTION
The patient is a 14-year-old G1P0 who presents to a private clinic requesting termination
of pregnancy. Her last menstrual period began 9 weeks prior to arrival. She has been
experiencing intermittent nausea and vomiting. She is sexually active with her 21-yearold partner and claims she is having consensual sex with him. She has been using
condoms for contraception. She denies past treatment or symptoms of STDs.
PMH: asthma diagnosed at age 3, requires no treatment
PSH: none
Medications: none
NKDA
Family history
Noncontributory
Social history
“Social” alcohol (3-5 drinks a day on weekends), no IVDA or tobacco use. Lives with
mother and 17-year-old brother
ROS
Negative
Physical Exam
P: 68, RR: 16, BP: 100/60, T = 37.6
Weight: 120 pounds
Abdomen: no hepatosplenomegaly, soft non tender
Pelvis: Uterus 8 wks, no adnexal masses or tenderness
Laboratory
UCG: positive
Pending: HIV, RPR, Pap smear
Ultrasound: 8 week IUP with fetal heart motion
Assessment/Plan
Patient is a 14-year-old at 8 weeks who desires termination of this pregnancy. Patient is
at risk for sexually transmitted diseases, and should also be assessed for the possibility of
sexual abuse. The patient is a candidate for either medical or surgical abortion.
Teaching Points
1.
2.
3.
4.
5.
6.
Options counseling: carrying the pregnancy, adoption, abortion
Consent issues:
a. States have different definitions of sexual coercion/rape. In some states, a
5 year discrepancy in age (such as in this case) for a girl under 16 years
old constitutes statutory rape and would need to be reported. It is
important to have access to state laws for sexually active young minors.
b. States have different regulations regarding parental notification/consent of
abortion for minors. Some states have no notification/consent laws: an
abortion can occur at the request of the minor. In some states, one parent
or legal guardian must be notified by phone or in writing and/or consent to
the procedure. It is important to have access to state laws for minors who
request abortion.
Abortion: Techniques of both medical and surgical abortion and their
advantages and disadvantages should be reviewed.
Contraceptive counseling: The patient should be counseled about
contraception, and should be encouraged to use condoms consistently in
addition to the contraceptive of her choice.
Self-esteem/education counseling: An evaluation of this patient’s social
situation and the reasons for her having a sexual relationship with a 21-year
old should be explored.
STD testing: A Pap smear and STD testing should be considered
References
Cook RJ, Dickens BM. Human rights and abortion laws. International Journal of
Gynecology and Obstetrics 65 (1999) 81-87.
FIGO Committee for the Ethical Aspects of Human Reproduction and Women’s Health.
Ethical guidelines on human reproduction and women’s health. Journal of Reproductive
Medicine 64 (1999) 317-322.
American Medical Association (AMA) Council on Ethical and Judicial Affairs. Code of
Medical Ethics. Chicago: American Medical Association, 1996.
CLINICAL CASE
POSTPARTUM HEMORRHAGE
At the conclusion of this exercise the student will be able to
A. Describe risk factors for postpartum hemorrhage
B. Describe the differential diagnosis of postpartum hemorrhage
C. Understand the management of the patient with postpartum hemorrhage
Renee is a 33-year-old G3P2 patient who is scheduled for an elective repeat cesarean
delivery. She had a cesarean section two years ago for fetal intolerance to labor and has
been counseled about vaginal birth after cesarean section. She was ambivalent about
another surgery but, ultimately, decided for this since she also wanted a tubal ligation and
this could be done at the same time. She presents to labor and delivery at 38 wk. gestation
with spontaneous rupture of membranes an hour earlier. She is experiencing intermittent,
moderately painful contractions. She has called her husband at work, but he has not yet
arrived. She doesn’t want to proceed to a cesarean delivery without him beside her.
Physical examination
Renee is quite uncomfortable with contractions that an external monitor reveals to be
occurring every three to four minutes. The fetal heart tones are in the 130s with
reassuring short and long-term variability. Her blood pressure is 110/80, pulse 80/minute,
respirations 20/minute. The remainder of the physical examination is unremarkable.
However, you notice clear fluid emerging from the vagina and a speculum exam reveals
fluid pouring from the cervical os. Her most recent cervical examination, yesterday in the
office, revealed 2cm dilation, 50% effacement and a fetal presentation high in the pelvis.
Because of the ruptured membranes, you are concerned about the possibility of a cord
prolapse and you re-examine the cervix, finding that it has changed to 4 cm dilation, 80%
effacement with a vertex presentation at –3 station. When given this information, Renee
states that she prefers to avoid a cesarean delivery and wishes to proceed with a vaginal
delivery, if possible. Renee’s husband arrives and is relieved to discover that she has
declined a repeat cesarean section. Labor progresses normally and, after pushing for
about an hour, she delivers a male infant with Apgar 8/9 weighing 7 pounds 13 ounces.
The placenta delivers about 15 minutes after the baby and the perineum appears intact.
Thirty minutes later you are called to the recovery room because Renee has experienced
brisk vaginal bleeding that did not respond to uterine massage by her nurse.
Laboratory
Per hospital protocol for previous cesarean section patients, a blood sample is sent for a
type and crossmatch, a CBC reveals WBC 8.8, hematocrit 37%, hemoglobin 9 and
platelets 280,000. A urine dipstick is negative for sugar, ketones or protein.
Diagnosis
Postpartum hemorrhage
Assessment/plan
You order Pitocin to be placed in the IV solution and check her blood pressure, which is
100/65. You ask that Methergine .20 mg be given intramuscularly. This is done but the
uterus continues to bleed. You return her to the delivery room where bimanual
exploration of the uterus is done after her epidural is re-dosed. You discover a fragment
of retained placenta and remove it manually. There is no evidence of a dehiscence of the
prior uterine incision. Gentle curettage of the uterine cavity fails to produce further tissue.
Careful vaginal exam reveals no cervical or vaginal lacerations. The bleeding slows
markedly, but is still a trickle; the uterus is moderately firm. You ask that a dose of
PGF2a be given intramuscularly and this is done. Further uterine massage succeeds in
reducing the bleeding to scant amounts. You order a CBC to be done in 4 hours and for
her intravenous line to be maintained. She is taken, once again, to the recovery room for
observation. Her blood pressure is stable, she is afebrile and she is alert, oriented and
comfortable.
Teaching points
1.
2.
3.
4.
Postpartum hemorrhage is defined as excessive bleeding in the
puerperium, after delivery of the baby and placenta. For vaginal deliveries,
this means over 500cc of blood loss; while in cesarean sections, the blood
loss should be over 1000cc to qualify as a postpartum hemorrhage.
Uterine atony is the most common cause and is, itself, caused by
precipitous labor; an overly distended uterus caused by such things as
macrosomia, multiple gestations or hydramnios; general anesthesia with
consequent uterine relaxation; amnionitis; oxytocin use in labor, especially
with prolonged inductions.
Lacerations of the vagina, cervix, vulva or perineum can lead to
significant blood loss and are particularly common in instrumented
deliveries (forceps, vacuum), macrosomia or precipitous labor where the
tissues have no time to stretch.
Retained placenta can result from a friable placenta, common in post dates
gestations; succenturiate placental lobes; prior cesarean section or prior
uterine curettage.
5.
6.
7.
Less common causes include coagulation defects, amniotic fluid
embolism, uterine inversion or hematomas in the vagina or vulva.
Management includes evaluation for all items in the differential diagnosis:
inspection for lacerations, treatment of an atonic uterus with massage,
oxytocin, Methergine, prostaglandin. If medical maneuvers fail, surgical
procedures can be employed. These include uterine artery ligation,
hypogastric artery ligation, uterine compression sutures, selective arterial
embolization or hysterectomy.
Circulation must be maintained with 2 large bore IV sites, IV fluid
infusion, and blood typing and crossmatch. Serial hematocrit will guide
plans for transfusion. Evaluation of coagulopathy may be needed if
bleeding persists, with the administration of whole blood or packed red
blood cells plus fresh frozen plasma or cryoprecipitate and platelets, if
necessary.
CLINICAL CASE
AMENORRHEA
A 26-year-old G2P2 woman with LMP= 6 months ago presents with a concern
regarding no periods. She delivered two full term healthy children vaginally and their
ages are 5 and 3. She stopped breastfeeding 2 years ago. She has noted a persistent
breast discharge, but no breast masses. She is not using any contraception, but
parenting has taken a toll on the husband-wife relationship and they infrequently find
the opportunity to have intercourse.
Ob-gyn history
G2 P2. 2 full-term vaginal deliveries of a 6-0 pound girl and a 7-8-pound boy. Pap
smears are up-to-date and normal. No STDs.
Past medical history
Postpartum depression, which resolved after one year on an SSRI.
Past surgical history
Cholecystectomy after her first pregnancy
Social history
Nonsmoker. Occasional alcohol. No street drugs. Married. Works as a housewife.
Family history
Noncontributory.
ROS
Increased stress since the delivery of the second child. Occasional hot flashes. Fatigue.
Headaches. Difficulty losing the pregnancy weight gain.
Physical exam
VS: BP= 120/80, P= 64, R= 18, Ht= 5’8”, Wt= 160 pounds
General: tired appearing Caucasian woman in no apparent distress
HEENT: NC/AT
Neck: No thyromegaly palpable
Lungs: clear
CV: Regular rate, no murmurs
Breasts: bilateral milky white discharge with expression. No masses, dimpling or
retraction
Abdomen: non-tender, no distension, no masses, no hepatosplenomegaly
Ext: Non-tender, no edema, DTRs 1+/= bilaterally
Pelvic exam: Normal external genitalia, moist vagina with decreased rugae, no
discharge, Cervix is multipara, non-tender, and no lesions, uterus is non-tender,
mobile and normal size, adnexae are non-tender and no palpable masses
Allergies: None
Medications: Multi-vitamin
Laboratory/studies:
HCG= negative
FSH= 3.5 mIU/mL
TSH= 2.5 uIU/mL
Prolactin= 130 ng/mL; repeat on fasting, 100ng/mL
Breast discharge smear reveals multiple fat droplets
MRI of the head reveals a 0.8 cm mass in the anterior pituitary
Diagnoses
Amenorrhea
Galactorrhea
Prolactinoma (Pituitary microadenoma)
Treatment
This patient was treated with Cabergoline (a dopamine agonist) on a weekly basis and
the dose was increased until her prolactin level was in the normal range. She tolerated
the medication well. She had return of menses within a few months time. Her
galactorrhea slowly resolved. She is now being followed on an annual basis.
Teaching points
1. There are multiple causes of amenorrhea and the student should become
familiar with them. This patient with her symptomatology could have easily
have been pregnant, had hypothyroid disease, premature ovarian failure or
2.
3.
4.
5.
hypothalamic amenorrhea associated with stress. It is important to consider
the entire differential diagnoses list prior to treatment.
Prolactinomas are the most frequent pituitary tumor and these
microadenomas tend to have an indolent course.
The elevated prolactin levels produce amenorrhea by inhibiting the pulsatile
secretion of GnRH and result in low gonadotropins and estrogen levels.
It is important to also evaluate both the TSH and prolactin levels in these
patients. Hypothyroidism may be present with increased prolactin levels since
TRH can stimulate both TSH and prolactin secretion.
Only 1/3 of women with high prolactin levels will have galactorrhea.
CASE STUDY REPRODUCTIVE GENETICS •
•
•
•
•
•
•
•
What is Sequential screen? What is QUAD screen? What does MSAFP stand for? What is the utility of raised MSAFP? What do you know about ONTD, Omphalocele and Gastroschisis? What is the usefulness of genetic amniocentesis? What is CVS? Look at the ACOG genetic screen questions in the ACOG prenatal records that are used in clinics…you understand all the conditions questions are being asked about? CLINICAL CASE
CERVICAL DISEASE AND NEOPLASIA
Did you know PAP no longer required until age 21yrs…Why?
S.K. is a 29-year-old G 3 P 2012 with a last menstrual period 12 days ago presenting
today for her routine gynecologic exam and refill of her birth control pills.
Ob-Gyn history
Menarche age 13
Periods every 26-29 days, 3-4 day flow, 4-6 pads per day
G3P2012 Full term NSVD x 2 without complications
Elective termination at 8 weeks, no complications
Last Pap smear 1 year ago, no abnormal Pap smears
Chlamydia at age 21, treated as a outpatient, no PID
Vulvar warts treated with trichloroacetic acid (TCA) 3 years ago
Uses birth control pills, used condoms in the past
Medical history
Recurrent otitis as a child Urinary tract infections x 2 over lifetime
Surgical history
Pressure equalization tubes, age 2, no complications
Medications
Combination oral contraceptives x 3 years
Allergies
Sulfa
Family History
Mother alive at age 62, had breast cancer at age 58; father deceased of myocardial
infarction at age 63
Social history
Smokes 1 pack of cigarettes per week
Drinks a glass of wine 1-2 times per week
No illicit drugs
Married x 7 years
Lives with her husband at two children, ages 6 and 5
Review of symptoms
She reports some “bumps” on her vulva x 1-2 months. They are not bothering her
occasional post-coital spotting.
Physical exam
Vital Signs: BP: 110/70
Weight: 135
Height: 65 inches
General:
Pleasant, well-nourished female who looks her stated age
BREASTS: Symmetric, no dominant masses, skin changes or nipple discharge
ABD:
Soft, non-distended, non-tender, no organomegaly
External Genitalia: Two raised lesions between her labium minus and majus on the
left hand side; edges irregular, but distinct; non-tender, normopigmented; not bleeding; maximum diameter 5 mm.
Vagina:
No discharge or bleeding; no visible lesions.
Cervix:
Multiparous, 5 mm endocervical polyp visible, no other lesions, no
cervical motion tenderness.
Uterus:
Anteverted, small, smooth, mobile, non-tender
Adnexa:
No masses, non-tender
Rectovaginal: Normal sphincter tone, no masses
Bladder:
Non-tender
Questions
1. At baseline, what are the general recommendations for routine Pap smear
screening?
2. What are this patient’s stated risk factors for cervical dysplasia?
3. What other risk factors could you ask about?
4. You diagnose the patient with vulvar condylomata, for which she declines TCA
treatment due to symptoms of severe burning the last time she was treated. What are
her other management options? Which HPV types are associated with vulvar
condylomata?
As the warts are asymptomatic, she decides to wait and see if they will regress
spontaneously. You encourage her to quit smoking, send her Pap smear to the lab
and refill her birth control pills.
5. Two weeks later, her Pap smear report indicates that your specimen was “adequate
for interpretation.” In general terms, what does this mean?
The Pap smear is interpreted as a low-grade squamous intraepithelial lesion (LGSIL).
What is the next step in her management? Would HPV typing be useful in this
situation?
6. The patient returns to your office after her next period for a colposcopic evaluation.
She reports that the vulvar condylomata have spread and she is reconsidering
treatment. Inspection of her vulva reveals two new warts on her right labium majus
measuring approximately 3 mm each.
What agents are used for a colposcopic exam of the cervix and why do they aid in the
management of abnormal Pap smears?
What are the features you look for after acetic acid is placed on the cervix?
What are the features you look for after Lugol’s iodine is placed on the cervix?
7. Your colposcopic evaluation is “adequate.” What does this mean?
8. At the 12 o’clock position on her cervix, you identify an acetowhite lesion
containing vascular changes resembling a mosaic pattern. You biopsy this lesion and
obtain an endocervical curettage. The pathology report reads:
Ectocervical biopsy:
Endocervical curettage:
Cervical intraepithelial neoplasia – grade 2
Normal endocervical glands, no dysplasia noted
What are your management options at this juncture?
9. After counseling the patient, you decide to treat her cervix with a LEEP and to
treat her vulvar condylomata with imiquimod.
10. The final LEEP pathology specimen reveals cervical intraepithelial neoplasia –
grade 2. All surgical margins are negative for dysplasia. What would you recommend
for follow-up?
CLINICAL CASE
INFERTILITY
A 37-year-old female and her 37-year-old husband present with the complaint of a
possible fertility problem. The couple has been married for 2 years. The patient has a 4year-old daughter from a previous relationship. The patient used birth control pills until
one-and-a- half-years-ago. The couple has been trying to conceive since then and report a
high degree of stress related to their lack of success.
The patient reports good health and no problems in conceiving her previous pregnancy or
in the vaginal delivery of her daughter. She reports that her periods were regular on the
birth control pill, but have been irregular since she discontinued taking them. She reports
having periods every 5-7 weeks. Past history is remarkable only for mild depression.
Imipramine 150 mg qhs for the last 8 months is her only medication. She works as a
cashier, runs 12-24 miles each week for the last 2 years, and has no history of STDs,
abnormal Paps, smoking, alcohol or other drugs. She has had no surgery.
The patient’s partner also reports good health and reports no problems with erection,
ejaculation or pain with intercourse. He has had no prior urogenital infections or exposure
to STDs. He has had unprotected sex prior to his current relationship, but has not
knowingly conceived. He has no medical problems or past surgery. He works as a longdistance truck driver and is on the road 2-3 weeks each month. He smokes a pack of
cigarettes a day since age 18 and drinks 2-3 cans of beer 3-4 times a week when he’s not
driving. He occasionally uses amphetamines to stay awake while driving at night. The
couple has vaginal intercourse 3-5 times per week when he is at home.
Physical exam
The patient is 5’9” and weighs 130 pounds. Breast exam reveals no tenderness or masses,
but bilateral galactorrhea on compression of the areola. Pelvic exam reveals normal
genitalia, a well-estrogenized vaginal vault mucosa and cervical mucus consistent with
the proliferative phase. The uterus is anteflexed and normal in size without masses or
tenderness.
Laboratory
Patient
TSH
Free T4
Prolactin
FSH
LH
Results
2.1 mIU/ml
1.1 ng/dl
60 ng/ml
6 mIU/ml
4 mIU/ml
Normal Values
0.5-4.0 mIU/ml
0.8-1.8 ng/dl
<20 ng/ml
5-25 mIU/ml
5-25 mIU/ml
Basal body temperature chart shows a monophasic temperature graph.
Partner
Semen analysis revealed 2 cc of semen, 4 million per mL, 30% normal forms and 20%
motility.
Management
The patient’s major infertility factor is anovulation; the most likely cause is
hyperprolactinemia from imipramine. The prolactin level is elevated, consistent with
drug-induced hyperprolactinemia. The patient was instructed, in conjunction with her
therapist, to taper off the imipramine. Her follow-up basal body temperature chart was
biphasic, consistent with ovulatory cycles.
The patient’s partner, however, has a semen analysis that is consistent with oligospermia.
The couple was given their options of: 1) In Vitro fertilization with ISCI; 2) artificial
insemination with partner’s sperm; 3) artificial insemination with donor sperm; or 4)
adoption.
Teaching points
1. Multiple causes must be considered for infertility diagnosis and treatment. Female
and male reproductive anatomy and physiology should be reviewed in order to
encompass the large differential diagnosis and the different factors that may
contribute to infertility or subfertility.
2. Infertility is defined as one year of unprotected coitus without conception.
Infertility may be primary: a woman who has never achieved pregnancy, or
secondary: a woman who has achieved pregnancy in the past. Causes of infertility
include tubal and pelvic pathology (35%), male problems (35%), ovulatory
dysfunctional (15%), unexplained infertility (10%) and unusual problems (10%).
3. Components of an initial infertility workup include a thorough history and
physical examination. Laboratory investigations include a semen analysis to
assess male causes of infertility, a method to document that ovulatory cycles are
occurring and, often, a hysterosalpingogram to rule out tubal disease.
4. Disorders of ovulation include polycystic ovarian syndrome (PCOS),
hyperprolactinoma, thyroid dysfunction and hypothalamic causes and may be
diagnosed by history, as well as laboratory tests that document the occurrence of
ovulatory cycles. Basal body temperature recording, urinary LH testing,
endometrial biopsy and luteal phase serum progesterone testing are all tests that
may help confirm the presence or absence of ovulation.
5. Dysfunction of the hypothalamic-pituitary-ovarian axis and medical illness,
including thyroid disease and pituitary tumors, can cause ovulatory disturbances.
Further laboratory workup targeting these problems should be performed when
history, examination, and initial laboratory evaluation indicate ovulatory
dysfunction. Medication can also cause ovulatory problems.
6. Ovulatory dysfunction may sometimes be treated with correction of medical
disease or change of medications. Most often, ovulatory dysfunction is treated
empirically with ovulation induction agents such as clomiphene citrate. A
thorough understanding of the normal physiology and pathophysiology of the
menstrual cycle aids greatly in the understanding of ovulatory dysfunction.
7. Management of tubal disease is often surgical, including lysis of adhesions and
removal of tubal obstruction via either laparoscopy or laparotomy, depending on
the severity of the disease.
8. Male fertility problems include varicocele, duct obstruction, sperm antibodies,
hypogonadism, testicular hyperthermia, drug use and industrial pollutants.
CLINICAL CASES
SEXUALLY TRANSMITTED INFECTIONS (STIS)
& URINARY TRACT INFECTIONS (UTIS)
Case 1
A 26-year-old gravida 3 presents to the clinic complaining of growths on her vulva.
These growths were noted one week ago, but she may have had similar lesions 4 years
ago during her last pregnancy. Of concern to her are the size of the growths and the
fact they are on both sides of her perineum and vulva. She has not sought treatment
before today.
PMH
Medical – unremarkable
Surgical – Postpartum tubal ligation 4 years ago
Obstetric – 3 living children ages 10, 8, and 4
all uncomplicated vaginal deliveries
Physical examination
Abdomen: Soft, benign, no hepatosplenomegaly
Pelvic: NEED INFO HERE
Vulva: multiple small (4mm) and large (14mm) fleshy soft “wart like” lesions
on both right and left labia and vulva
Vagina: soft, moist and pink, no lesions
Cervix: multiparous, no gross visible lesions
Uterus: slightly enlarged, firm, mid-position, anteverted and anteflexed
AP
Condyloma acuminate with plans for patient applied therapy with Imiquimod.
Discussion
Human papillomavirus infections, although common, are frequently asymptomatic,
unrecognized and maybe subclinical. Overall, there are 70 viral subtypes of HPV.
HPV 6 and 11 are generally associated with visible genital warts, while viral types 6,
18, 31, 33 and 35, as well as 45 and 56, have been associated with cervical neoplasia.
Genital warts generally occur on the perineum and vulva as well as the perianal skin
but may be identified on the uterine cervix as well as the vagina, urethra, anal canal
and mouth. Clinically these lesions are described as fleshy and vary in size from very
small to very large. Classically, the lesions are described as soft fleshy growth and are
termed condyloma acuminata or venereal warts.
The diagnosis of condyloma acuminata is based not only on physical findings, but
may be confirmed by biopsy of the genital warts. Management options include
destruction of the lesions using either electrocautery laser or local incision, as well as
chemical destruction using podophyllotoxin or trichloroacetic acid. Other methods of
therapy include immunological means using interferon, intralesional or Imiquimod
patient-applied therapy. The current CDC recommendations are that the first line of
therapy includes patient applied therapy using Podofilox 5% solution or gel or
Imiquimod 5% cream. There is no evidence that would suggest that any of these
regimens are superior to the others and the treatment should be guided based on the
preference of the patient. Importantly, if the patient has not improved
symptomatically and the lesions begun resolving after 3 weeks therapy, the therapy
should be changed to provider administered treatment. In the case of women with
cervical warts, high-grade squamous intraepithelial lesions must be excluded before
treatment is initiated. Treatment for cervical warts and/or vaginal warts includes
cryotherapy or TCA. Follow up of women who have had genital warts includes
regular annual examinations and Pap smears.
Teaching points
HPV is a most common STD with more than 70 DNA subtypes.
HPV 6, 11 are associated with genital warts.
HPV 16, 18, 31, 33, 35, 45, 56 associated with cervical cancer.
Diagnosis based on clinical exam and/or biopsy.
Treatment involves physical destruction of lesions using electrocautery, laser, local
excision, or chemical destruction (Podophyllotoxin or trichloroacetic acid) or
immunological (Interferon or Imiquimod).
Case 2
A 16-year-old G1P1 where LMP was one week ago presents with a one-week history of
severe lower abdominal pain. Pain is constant, bilateral and accompanied by fever
and chills. She has had some nausea and several episodes of vomiting. She has been
sexually active for 3 years and has had unprotected intercourse with several partners.
She denies irregular bleeding, dysmenorrhea or dyspareunia.
PMH
Medical – negative except childhood illness
Surgical – Tonsillectomy as a child
Obstetrical – Gravida1, para1, uncomplicated vaginal delivery one year ago
Physical exam
Ill appearing 16-year-old
T 38.6o
P 94
BP 124/82
R 22
Abd: Bilateral lower abdominal tenderness, slightly distended with
rebound, negative psoas and Murphy’s signs
Pelvic: BUS negative
vag – pink, moist,
cervix - purulent discharge from os, indurated
uterus – midline position, soft, tender
adnexa – bilateral fullness and moderately tender
Labs
GC: positive
RPR: negative
WBC: 17.6 with left shift
UA: few WBC’s, no bacteria, 3 plus ketones
UHCG: negative
Diagnosis
Acute salpingitis with peritoneal findings. Admit for parenteral antibiotics.
Discussion
The most common presenting complaint of women who have PID is lower abdominal
pain. Associated symptoms include vaginal discharge, irregular bleeding,
dysmenorrhea, dyspareunia, dysuria, nausea, vomiting and fever. Women who have
gonococcal infection have evidence of more acute inflammation (peritoneal signs,
fever, leukocytosis) than those who have nongonococcal infection because of the
endotoxin produced by N gonorrhoeae. Most women who have acute PID present
during the first half of the menstrual cycle. Presentation later in the cycle indicates
an infection of longer duration and increases the likelihood that a TOA has organized.
Atypical presentations of PID are common and complicate the differential diagnosis.
For example, the symptoms of Fitz-Hugh-Curtis syndrome may mimic hepatitis or
cholecystitis.
All PID treatment regimens must provide broad-spectrum coverage of likely
pathogens, including N gonorrhoeae, C trachomatis, anaerobes, gram-negative bacteria
and streptococci. Although broad-spectrum antibiotic coverage results in
symptomatic improvement in most patients, the risk of long-term sequelae remains
high. Adolescents are at particularly high risk for future reproductive complications
because of their tendency not to complete prescribed treatment regimens. The CDC,
therefore, recommends serious consideration of hospitalization for all adolescents who
have PID. Hospitalization is especially recommended if compliance is unpredictable,
the diagnosis is uncertain, or pelvic abscess is suspected. Hospitalization is essential if
the patient is pregnant, has HIV infection or is too ill to tolerate or has failed to
respond to outpatient therapy. All patients who are managed as outpatients must be
reevaluated within 72 hours of the initiation of antibiotics.
Sex partners of patients with PID should be examined and treated if they had sexual
contact with the patient in the 60 days prior to the onset of symptoms. Partners
should be treated empirically for N gonorrhoeae and C trachomatis, regardless of the
apparent etiology of the PID or pathogens isolated from the infected woman.
Without treatment of infected partners, risk of reinfection is high.
Teaching points
Pelvic pain, fever and vaginal discharge are the most common findings if secondary to
gonococcal infection. Patients maybe asymptomatic if chlamydia is the
causative organism.
Salpingitis is usually secondary to gonorrhea or chlamydia infections or both.
Diagnostic gold standard is laparoscopy.
Treatment regimens consist of an inpatient or outpatient protocol.
Sequelae include chronic pelvic pain, ectopic pregnancy and infertility.
References
Centers for Disease Control and Prevention. Sexually Transmitted Disease Treatment
Guideline 2002. MMWR 2002; S1:48-52; www.cdc.gov/std/treatment/default.htm.
McCormack WM. “Pelvic inflammatory disease.” N Engl J Med. 1994;330:115-119.
Paavonen J. Dermatol Clin 1998 Oct; 16(4): 747-56, xii.
Pletcher JR. Pediatr Rev 1998 Nov; 19(11):363-7.
Smith M, Shimp L. Women’s Health Care, 20 Common Problems 2000, 433-6.
Case 3
Ms. Sandy R is a 32-year-old, G2P0020 female who comes into the ER having
experienced 8 hours of right lower quadrant pain.
The pain is sharp, stabbing and intermittent in nature and non-radiating to the back.
It is associated with 3-4 episodes of nausea and vomiting that preceded the pain. She
also complains of dizziness and has had loose bowel movements since the onset of the
pain. The pain increases with movement. Fetal position is least painful. She had a
similar episode of pain 10 years ago. A laparoscopy was performed at that time and
was negative for pathology.
Ms. R states that her last menstrual period ended yesterday. She denies being
sexually active currently. She denies fever, chills, dysuria, increased urinary
frequency, hematuria, melena, vaginal discharge, or vaginal bleeding or spotting. Ms
R has allergies to penicillin and no other known allergies. Ms. R is not taking any
medications currently. Menstrual history is menarche at age 13, cycles every 26 days
with 5 days of bleeding. She had a chlamydia infection 9 years ago that was treated
with antibiotics. Two years ago she had a vaginal monilia infection that was treated
with Monistat. Her last Pap smear (1 year ago) was normal. She has used oral
contraceptives intermittently for 8-10 years and last used them 2 years ago. Ms. R
has never used an intrauterine device. She is presently not using any type of birth
control method.
She denies having a history of hypertension, diabetes, heart disease, asthma or kidney
disease.
Surgical history
Tonsillectomy 17 years ago, laparoscopy 10 years ago and elective abortions without
complications 6 and 8 years ago.
Social history
Ms. R lives alone in an apartment and works as a sales clerk. She denies smoking,
drinks 1-2 beers on the weekend and denies use of recreational drugs. She has no
family history of hypertension, heart disease, asthma, kidney disease, diabetes or
bleeding disorders.
Review of symptoms
Unremarkable.
Physical examination
The physical examination reveals a 32-year-old female in no apparent distress. She is
alert and oriented x 3.
Vital Signs:
Temperature:
99.2° F (oral)
Blood pressure:
112/70 mm Hg, supine
116/72 mm Hg, sitting
Pulse:
96 beats/minute, supine
100 beats/minute, sitting
Respirations:
16 breaths/minute
Weight/height:
128lbs; 4’11”
HEENT:
Unremarkable
Neck Exam
Reveals no cervical lymphadenopathy or
thyromegaly
Lung Exam:
Clear
Heart Exam:
Heart tones normal without a murmur. She has
good peripheral pulses.
Abdomen:
Obese with positive bowel sounds and is tender to
palpation throughout; uterine tenderness greater
in the right lower quadrant. There is positive
guarding and rebound tenderness.
Pelvic Exam:
Reveals normal external genitalia, pink, rugose
vaginal walls and a closed os with green
discharge. The uterus and adnexa cannot be
evaluated completely secondary to pain.
Cervical motion tenderness cannot be ruled out.
Rectal Exam:
Guaiac negative and without masses.
Extremities:
Without cyanosis, clubbing, or edema. Few
superficial varicosities are seen on upper legs.
Neurologic Exam:
Grossly within normal limits.
On admission
WBC:
11.1
Hemoglobin:
14.3
Hematocrit:
41.7
Urinalysis:
Negative
Segs:
77
Bands:
18
Platelets:
Normal estimate
PT:
115%
PTT:
23 seconds
Pregnancy Test:
Negative
Hospital course
Ms. R. is prepared for the OR.
Surgical report
Ms. R. was anesthetized, placed in the dorsal lithotomy position, prepped and
draped in the usual manner. An infraumbilical incision was made and a Trocar was
inserted without complications. Vaginally, a single-toothed tenaculum was used to
grab the anterior lip of the cervix and a Hulka manipulator was placed. A
suprapubic incision was made and a second Trocar was inserted under direct
visualization.
Upon entering the peritoneum, free pus was encountered. The appendix was
inflamed, but it appeared to be a periappendicitis without evidence of any acute
perforation or intrinsic inflammation of the appendiceal wall. Some of the rest of the
serosa of the terminal ileum and cecum also appeared inflamed. Upon exploring Ms.
R a little more inferiorly, a pocket of pus is entered and approximately 100cc of
fairly liquid pus found; this was cultured. Upon inspection of the right tube and
ovary, there appeared to be a luteinized cyst in the right ovary that was in the
process of decompressing. The right tube had marked scarring with what appeared
to be a hydrosalpinx with fimbriae scarring at the end. On inspection of the left
tube, there was marked dilatation of the distal tube with fairly normal diameter,
approximately 1cm from the left cornu of the uterus. At this point, there was a
marked hydrosalpinx with a pyogenic component, with creamy pus emanating from
the area just by the fimbriae, which appeared almost necrotic. The left ovary was
not involved. The diagnosis of acute left salpingitis was made. The pelvis was
irrigated and the instruments were removed. The incision was closed with 3-0 Vicryl.
The patient was extubated and taken to the recovery room in stable condition.
Post-operative diagnosis
Peritonitis with acute salpingitis of the left tube. Hydrosalpinx of the right tube with
fimbria scarring. Peri-appendiceal inflammation.
Procedure
Diagnostic laparoscopy
Post-operative
Ms. R. continued to spike fevers. Her cultures from the OR revealed Escherichia coli,
Neisseria gonorrhoeae and Bacteroides fragilis. Blood cultures were subsequently
drawn during one of the temperature spikes, which revealed E. coli. Eventually, 3
days post-operatively, Ms. R. developed induration and redness at the wound site.
The sutures were removed and the wound was packed open for the duration of her
hospital stay. Ms. R. was maintained on Gentamicin and Cleocin for 5 days postoperatively. She had Gentamicin levels drawn, which were in the therapeutic range.
The organisms which grew from the wound, blood and abscess were sensitive to
Gentamicin. The Neisseria is penicillin resistant. Ms. R. eventually defervesced
several days prior to discharge. The wound continued to improve throughout her
hospital stay. The induration and redness had dissipated by the time of discharge.
The antibiotics were switched to Ciprofloxacin in anticipation of discharge; however,
Ms. Rose developed a rash that was pruritic in nature. This was treated with
Benadryl. The Ciprofloxacin was stopped and the rash resolved. She remained afebrile
and was discharged home on Benadryl 50 mg po q.i.d., PRN, Doxycycline 200 mg po
b.i.d. x 7 days and Tylenol #3, 1-2 tabs po q3-4 hours PRN pain.
Ms. R. was given instructions for wound care and a clinic appointment for follow-up
in 2 weeks.
CLINICAL CASE
OVARIAN NEOPLASMS
A 57-year-old multiparous woman presents with a complaint of progressive fatigue
over a period of six months. She also complains of increasing abdominal girth and
dyspepsia, for which she is taking over the counter antacids. Her gynecological
history is unremarkable. She underwent menopause at age 51 and chose to take no
hormone replacement therapy. Her last Pap smear and pelvic exam was three years
ago and was normal.
Physical Examination
Physical examination reveals a 57-year-old white female who is mildly short of
breath. Patient blood pressure is 122/80. HEENT examination and examination of
the heart and lungs is normal.
On abdominal examination, the patient has a large irregular pelvic mass which
extends nearly to the xiphoid process.
On pelvic examination, the vagina and cervix are normal. The pelvis is filled with a
mass that appears to be contiguous with the abdominal mass.
Laboratory
Hemoglobin 10.2, Hematocrit 33.4. Indices are normal.
Chest X-Ray is normal; there are no pleural effusions.
Pelvic ultrasound reveals a large mass with multiple projections. Ascites is present.
Some areas of the mass appear cystic, where others appear solid. The mass fills nearly
the entire abdomen.
Diagnosis
Ovarian carcinoma. The large size of the tumor makes it likely to be mucinous or
serous cystadenoma/cystadenocarcinoma.
Management
The patient is taken to the operating room for laparotomy for diagnosis and cyto
reduction, if malignant. There are multiple “seedings” of tumor. Peritoneal washings
are not done because gross tumor deposits greater than 3cm are present throughout
the peritoneal cavity.
Because of the 3C stage of the tumor and the presence of small amounts of tumor
following cytoreductive surgery, the patient is started on a course of chemotherapy
utilizing paclitaxel and carboplatin. After a complete course of chemotherapy, she
seems to be doing well.
However, six months following chemotherapy, she presents to the emergency room
with a complete small bowel obstruction. She undergoes a small bowel resection with
removal of recurrent ovarian tumor. She receives another course of chemotherapy;
however, her condition continues to deteriorate. She dies 48 months after initial
diagnosis
Teaching Tips
1.
2.
3.
4.
5.
6.
“Although diverse ovarian tumors generally manifest in a similar
manner, the diagnosis of early ovarian cancer is more a matter of
chance than a triumph of the scientific method.” DiSaia1 pp. 296(See
Objective.)
Mortality rate from ovarian carcinoma is the highest of all
gynecological malignancies.
Malignant ovarian tumors are categorized by cell type. 90% are
epithelial in origin. They spread primarily by exfoliation.
Most women have advanced disease at the time of diagnosis.
Early diagnosis is difficult because of lack of effective screening tests.
Cytoreductive surgery (tumor debulking) is the mainstay of therapy.
Adjuvant chemotherapy is frequently used because of the advanced
stage of disease at the time of diagnosis.
CLINICAL CASE
What are the risk factors for uterine cancer?
What the various cell types of uterine cancer?
Know the staging and treatment for uterine cancer
GESTATIONAL TROPHOBLASTIC NEOPLASIA
A 15-year-old primigravida presents for routine prenatal care. She is 14 weeks pregnant,
and complains of some nausea but, otherwise, feels well. The pregnancy to date has been
unremarkable. She does have support from her parents and the father of the baby.
Physical examination
The uterus is enlarged, measuring 27 cm from the pubic symphysis. Fetal heart tones are
not auscultated. She denies vaginal bleeding or passage of tissue from the vagina.
Vaginal exam is unremarkable.
Laboratory
Routine prenatal labs were unremarkable. She is Rh-positive. Quantitative beta HCG
levels were markedly elevated at 112,320 MICU/ml. TSH was low and further thyroid
testing revealed the patient to be mildly hyperthyroid.
Ultrasound is ordered at this visit, which shows the uterus to be enlarged, with multiple
internal echoes and a “snow storm” appearance. No fetus is noted. It also shows enlarged
multiloculated ovarian cysts bilaterally.
Chest x-ray was normal.
Diagnosis
Hydatidiform mole with bilateral theca lutein cyst
Management
Patient underwent a suction evacuation of the uterus. There were no surgical
complications. The patient was placed on oral contraceptives. Beta HCG levels were
obtained weekly and remained elevated. Chest x-ray was negative. The patient was given
a course of methotrexate. The beta HCG levels dropped to undetectable levels upon
completion of the chemotherapy. The thyroid function normalized. Chest x-rays remained
negative. Pelvic examination and ultrasound showed resolution of the bilateral ovarian
cysts. She became pregnant 16 months after treatment and delivered a healthy infant at
term.
Teaching points
1.
2.
3.
4.
5.
6.
Gestational trophoblastic neoplasia is the most curable gynecological
malignancy.
Although patients with hydatidiform mole are classically described as having
uteruses that are large for dates, this only occurs in approximately half of the
patients.
Molar pregnancies are more likely to occur in women 15-years-old or less, or
40-years-old and greater.
Gestational trophoblastic neoplasia is frequently associated with
hyperthyroidism due to the release of thyrotropin-like compound by the molar
tissue.
Although it is true that patients with molar pregnancy have increased risk of
trophoblastic disease in later pregnancies, recurrent trophoblastic disease only
recurs 1% of the time.
Patients with gestational trophoblastic neoplasia should not attempt
subsequent pregnancy until after six months of negative beta HCG levels.
CLINICAL CASE
MULTIFETAL GESTATION
At the conclusion of this exercise, the student will be able to:
1.
2.
3.
Discuss the etiology of multiple gestation
Discuss the zygosity of twin gestation, understanding diagnosis and
complications unique to each
Discuss antepartum, intrapartum and postpartum management and
complications of twin gestation.
A 32-year-old African-American nullipara at 33 weeks’ gestation, who has been
followed in your practice with a known twin gestation, presents for an office visit. She
has no known medical problems. Her pregnancy was the result of ovulation induction
and, other than having twins and anemia, has been uncomplicated to date. She has
had normal weight gain during the pregnancy and her BP and urine dipstick are
normal today. She complains of decreased fetal movement for both babies and her
fundal height is smaller than expected for this gestation. She also complains of
abdominal cramping, but denies leaking fluid. Fetal heart tones are documented for
both babies. Her last fetal ultrasound was five weeks ago and the technician thought
she saw a dividing membrane between the two babies. The estimated fetal weight at
that time was appropriate for gestational age and the babies were similar in size, with
the upper twin slightly smaller.
Ultrasound evaluation reveals normal cardiac activity for both twins; however, there
is a 28% difference in weights with Twin A (presenting, vertex) larger than Twin B
(upper, breech). There is normal amniotic fluid volume and a thin dividing membrane
is seen. An NST is performed and both twins have a reactive fetal heart rate pattern
with no decelerations. However, she is noted to be contracting every 3 minutes on the
tocodynamometer. Vaginal examination showed the cervix to be 3 cm dilated and
75% effaced with the vertex of Twin A at –1 station. The membranes appeared to be
intact.
She is admitted to labor and delivery and treated for preterm labor. Her contractions
do not slow down and she progresses to complete dilation. She is taken to the
operating room and Twin A delivers by spontaneous vaginal delivery without
complications. Twin B is delivered by total breech extraction and also does well. After
delivery, she does well and is discharged home after postpartum day #2.
Teaching points
1.
2.
3.
4.
Etiology of multiple gestation
Preterm labor and its associated morbidities
Utility of tocolytic medications
Delivery of twins
CLINICAL CASE
MEDICAL AND SURGICAL CONDITIONS IN PREGNANCY
A 22-year-old female presents to the emergency department with a 3 hour history of
steadily increasing lower abdominal pain. She has not seen a physician in over 3 years.
She has no history of medical or surgical problems and takes no medications. She
noted the pain shortly after eating lunch, and has had some nausea, but no vomiting.
She denies fever, dysuria, flank pain, and vaginal bleeding. She does not remember
her last menstrual period, because they are very irregular. She is sexually active, and
does not use contraception regularly. She was treated for a “pelvic infection” with an
injection one year ago, but she cannot recall the name of the infection. She denies use
of illicit drugs, but admits to alcohol abuse (one 6-pack daily and more on the
weekend) and gives a 6-pack year history of tobacco use. She describes the pain as a
gradual onset, becoming sharp and intermittent with waves of pain alternating with
short episodes of relief. An antacid did not give relief.
Physical examination reveals an anxious female lying on the exam table with hips
flexed, moaning softly. She intermittently cries and complains of increasing pain. The
nurse tells you her temperature is 99 degrees, and her pregnancy test is positive. Your
examination of her abdomen reveals a gravid uterus to the level of the umbilicus with
guarding and mild rebound tenderness in both upper quadrants. The uterus is soft and
non-tender to palpation. Pelvic examination reveals no amniotic fluid in the vagina,
and cervix is closed with no blood visible at the external os.
What tests/labs should be ordered at this time?
Urinalysis reveals 2+ glucosuria with infrequent WBCs and bacteria. CBC reveals a
WBC of 14,000 with no left shift. Ultrasound reveals an intrauterine pregnancy with
estimated gestational age of 22 weeks. Amniotic fluid volume appears normal.
What is your initial assessment and plan?
This patient has multiple potential causes of her pain. She should be evaluated for
both pregnancy-related and non-pregnancy related sources of her pain. The multitude
of possible causes frequently results in the need for consultation jointly by the
obstetrician and surgeon, and when surgery is indicated, both may need to be present
in the OR until the etiology has been determined.
Teaching/clinical points
1.
2.
3.
4.
Medical and surgical conditions may present differently during
pregnancy.
Pregnancy may impact the course and prognosis of certain medical and
surgical diagnoses.
The physiological changes that normally occur during pregnancy
MUST be considered when evaluating a gravida with a medical or
surgical problem to determine when clinical or lab findings are a normal
change of pregnancy and when they are indicative of disease.
Consultation with an internist or surgeon is frequently indicated when
complicated medical or surgical situations occur during pregnancy.
CLINICAL CASE
PRETERM LABOR
A 20-year-old African-American, who is 28 weeks pregnant, presents to the labor unit
complaining of contractions. The contractions began 8 hours ago and have increased in
frequency and duration. She notes that, for the last two days, she has had increased
vaginal discharge and some lower back pain. Her prior pregnancy was complicated by
preterm labor and premature ruptured membranes at 26 weeks gestation. The neonates
course was complicated by intra-ventricular hemorrhage and necrotizing enterocolitis.
PMH
Medical
Surgical
Obstetric
neg
appendectomy age 11
gravida 3 para 2 one preterm delivery, living children 2, one with
cerebral palsy
Physical examination
Vital Signs
kg
General
Cardiac
Abdomen
Vaginal exam
Temp 36.5oC (97.8oF), pulse 64, respiration 20, BP 100/60, wt 49
Thin women of stated gestational age
Regular rate and rhythm, no rubs, gallops or clicks
No hepatosplenomegaly, fundal height 27 cm
No fluid per os; cervix – 3cm dilated and completely effaced;
vertex presentation
AP
Preterm labor at 27 weeks gestation plan for tocolysis, steroids and penicillin for Group B
streptococcus prophylaxis
Discussion
Preterm birth is one of the major health hazards of our time. It is the leading cause of
neonatal morbidity and mortality. Approximately 11-12% of births occur prior to 37 wk.
gestation. Roughly 1/3 is due to preterm labor, 1/3 to preterm premature ruptured fetal
membranes and 1/3 to medical or fetal complications. Efforts to reduce or prevent
preterm birth are largely unsuccessful.
Preterm birth is defined as that occurring less than 37 wk. in the presence of regular
uterine contractions (4 per 20 min or 8 per 60 minutes) with cervical change or if the
cervix is 2 cm dilated and 80% effaced. There are many possible causes of preterm labor,
including infection, faulty placentation, uterine factors (leiomyomata, uterine didelphys),
overdistension of the uterus (multifetal gestation, polyhydramnios), immunologic causes,
drug use (such as cocaine), and idiopathic. Risk factors for preterm birth can be
subdivided into pre-pregnancy (i.e. low maternal weight and prior preterm birth) and
pregnancy-related (twins, abnormal placentation, maternal factors).
Management of women with preterm labor involves tocolytics to reduce or stop the
contractions (evidence is weak that tocolytics works longer than 24-48 hours),
corticosteroids to enhance lung maturation and decrease the likelihood of neonatal
respiratory distress syndrome and penicillin intrapartum to prevent early onset neonatal
GBS infection.
Tocolytics can be divided into major categories with various degrees of efficacy, safety,
side effect profiles, costs, etc. Categories include B-sympathomimetic (ritodrine and
terbutaline), magnesium sulfate, prostaglandin synthetase inhibitors (indomethacin),
calcium channel blockers (nifedipine) and oxytocin inhibitors (atosiban).
Teaching points
-
Preterm birth is a common pregnancy complication
Risk factors for preterm labor/birth are many; however the most common
risk factors are prior preterm birth and low maternal weight
Diagnosis of preterm labor is difficult
Management includes hydration, tocolytics (a high rate of failure to
prevent preterm birth), corticosteroids and antibiotics
________________________________________________________________________
CLINICAL CASE
PREMATURE RUPTURE OF MEMBRANES
A 29-year-old infertility patient with a triplet gestation at 29 weeks gestation presents to
the labor and delivery unit of your hospital describing a sensation of “leaking fluid from
the vagina” for the last 30-45 minutes. She says she noted her underclothes were damp
yesterday, and this morning she noted clear fluid with a small amount of blood leak from
her vagina after voiding and wiping her perineum. She denies fever, contractions and
abdominal pain, and feels the babies are moving normally.
Physical examination reveals an afebrile, alert, anxious female. Abdominal examination
reveals a 32-centimeter fundal height and a non-tender abdomen and uterus.
How should you further assess this patient?
Teaching points
Why is preterm birth risky?
Survival by gestational age*
Survival by birth weight*
<23 weeks
23 weeks
24 weeks
25 weeks
<500g
500-599g
600-699g
700-799g
0-12%
2-36%
17-56%
35-85%
1-38%
4-37%
27-63%
43-88%
* represents livebirths
Hack and Fanaroff; Semin Neonatol 2000; 5:89-106
Discussion
In addition to the risks of prolonged rupture of membranes in a preterm gestation, the risk
of preterm birth makes PROM a difficult situation. When PROM occurs at a previable
gestation, a discussion should be held with the family reviewing the maternal risks of
infection against the fetal risks of significant morbidity and mortality during expectant
management. When PROM occurs at a preterm, but potentially viable, gestation,
discussion should ensue regarding the risk of fetal and maternal infection, as well as risks
of preterm birth. This will allow the family to understand the benefit of antibiotics,
steroids and expectant management. Careful monitoring of mother and fetus during
expectant management should be undertaken, and delivery considered when documented
or suspected lung maturity or signs of fetal infection, unrelieved fetal stress or advanced
labor are noted. Counseling after the delivery for the recurrence risk of PROM should
occur, and modifiable risk factors addressed.